Howdy, Stranger!

It looks like you're new here. If you want to get involved, click one of these buttons!

PT3.S4.Q24 -It has been claimed that an action is morally good

IheartmycatIheartmycat Core Member
edited October 2021 in Logical Reasoning 9 karma

Having a hard time understanding why A is wrong. The action was intended to cause harm thus necessitating it to be morally bad….right?

I know I’m wrong. I just need to know why.

help

Admin Note: Edited title. Please use the format: "PT#.S#.Q# - brief description of the question"

Comments

  • mattscrappymattscrappy Member
    138 karma

    The map out of the conditionality is "If harm was intended or forethought could have prevented harm, then an action is morally bad." BUT, we only know that this applies if there actually is harm. We don't know if an action intended to harm but doesn't is also morally bad. AC A specifies that the intended harm hard the opposite effect, and so, since no harm occurred, we can't prove that it was morally bad and therefore AC A must be wrong. It's very compelling, but it doesn't trigger the conditionality of the principle listed. Hope this helps.

  • Ashley2018-1Ashley2018-1 Alum Member
    2249 karma

    I personally wouldn't diagram this out and would just keep the two conditional statements in mind and go through the answer choices one by one. But oh geez louise, A is attractive lol. But the problem is is that the action, aka Pamela writing that letter attempting to sabotage Edward and his friend...backfired and produced good results for the two of them and so the second conditional doesn't apply.

  • drd5494-1-1drd5494-1-1 Core Member
    8 karma

    As others have said, the stimulus says that the morally bad claim only applies to "an action that harms others", so A doesn't work because it didn't do harm.

    The difficulty I had with the question was that the correct AC (E) doesn't explicitly say that "reasonable forethought" would have shown that the action of getting distracted while watching a three-year-old is likely to cause harm. That's not an unreasonable assumption to make, but it is nonetheless one you have to make for E to be correct. Given that the other choices were just wrong, it was the AC that I felt "most closely conforms" to the principle. If there were a choice similar to E that explicitly stated the assumption, maybe it would have been a better choice.

Sign In or Register to comment.